7
$\begingroup$

Let $\phi:\Bbb D^2\to\Bbb R^n$ be a continuous mapping of the 2-disc $\Bbb D^2$ that is injective on the boundary $\partial\Bbb D^2=\Bbb S^1$. Does its image contain an embedded disc with the same boundary? That is, is there an injective map $\psi:\Bbb D^2\to\Bbb R^n$ with $\phi(x)=\psi(x)$ for $x\in\partial \Bbb D^2$ and $\mathrm{im}(\psi)\subseteq\mathrm{im}(\phi)$? Does this depend on $n$? Is it at least true if $\phi$ is a PL map?

This is meant to be an analogy to how every curve between two points contains a non-self-crossing curve between the same points (at least I believe that this is true).

$\quad\quad$

$\endgroup$
7
  • $\begingroup$ If I understand you correctly, this is the Schoenflies theorem. $\endgroup$ Nov 12 at 0:40
  • 1
    $\begingroup$ @Oleg For $n=2$ you mean? This seems plausible. I would be most interested in $n=3,4$. $\endgroup$
    – M. Winter
    Nov 12 at 0:41
  • $\begingroup$ @ThomasKojar Hm, I do not immediately see the relation of my question to the Schoenflies problem for $n\ge 3$. Can you elaborate? I still think that the horned sphere (even a less wild version) might be a counterexample in 3D. $\endgroup$
    – M. Winter
    Nov 12 at 0:50
  • 2
    $\begingroup$ I missed that your map goes to $\mathbb{R}^n$. The answer is no for $n=3$ - attach a disk (necessary with self-intersections) to a non-trivial knot. I don't know the answer for $n>3$. $\endgroup$ Nov 12 at 1:22
  • $\begingroup$ @Oleg Yes, that should suffice as a counterexample. Thank you. It also yields a counterexample for $n\ge 4$ because we can just make $\phi$ map your counterexample to a 3-dimensional subspace, and since $\psi$ must be contained in $\phi$ we are back at the 3-dimensional problem. $\endgroup$
    – M. Winter
    Nov 12 at 1:25

1 Answer 1

6
$\begingroup$

The answer is "yes" for $n=2$ (that is the Schoenflies theorem). For $n>2$ the answer is "no". Consider a non-trivial knot $K$ in $\mathbb{R}^3$, and a continuous injective map $\partial\mathbb{D}^2\mapsto K$. Extend that map to a map $\mathbb{D}^2\to \mathbb{R}^3$. The resulting disk has a knot as a boundary, so it must have a self-intersection. As pointed out by M. Winter, that gives a counter-example for all $n\geq3$.

Another way to construct counter-examples is to consider an embedded disk with a single point of a self-intersection. For $n\ge 4$ one may consider a disk with a transversal self-intersection. For $n=3$ one may consider a disk that touches itself -- fold a piece of paper and pinch two halves together at one point.

$\endgroup$

Your Answer

By clicking “Post Your Answer”, you agree to our terms of service and acknowledge that you have read and understand our privacy policy and code of conduct.

Not the answer you're looking for? Browse other questions tagged or ask your own question.